ASAP PLSSS!!!!!!!!!!1!!!!!!!!!!!!!!!!!!!!!!!!!!!

Answers

Answer 1

Answer: i think its the last one bc if you multiply 9 and 3 in its gets you 18

Step-by-step explanation:

Answer 2

Answer:

V=(1/3)πr²h

v=

[tex] \frac{1}{3} \times {3}^{2} \times 9 \times \pi \\ = 27\pi[/tex]

27pi in³

27pi in³First option

Brainliest please~


Related Questions

sin x - cos x - 1/√2 = 0
Find the value of x ​

Answers

Answer:

Step-by-step explanation:

A. If x:y= 3:5, find = 4x + 5 : 6y -3​

Answers

Answer:

17 : 27

Step-by-step explanation:

x=3

y=5

4(3)+5 : 6(5)-3

= 12+5 : 30-3

= 17 : 27

Coordinate plane with quadrilaterals EFGH and E prime F prime G prime H prime at E 0 comma 1, F 1 comma 1, G 2 comma 0, H 0 comma 0, E prime negative 1 comma 2, F prime 1 comma 2, G prime 3 comma 0, and H prime negative 1 comma 0. F and H are connected by a segment, and F prime and H prime are also connected by a segment. Quadrilateral EFGH was dilated by a scale factor of 2 from the center (1, 0) to create E'F'G'H'. Which characteristic of dilations compares segment F'H' to segment FH

Answers

Answer:

[tex]|F'H'| = 2 * |FH|[/tex]

Step-by-step explanation:

Given

[tex]E = (0,1)[/tex]             [tex]E' = (-1,2)[/tex]

[tex]F = (1,1)[/tex]             [tex]F' = (1,2)[/tex]

[tex]G = (2,0)[/tex]             [tex]G' =(3,0)[/tex]

[tex]H = (0,0)[/tex]            [tex]H' = (-1,0)[/tex]

[tex](x,y) = (1,0)[/tex] -- center

[tex]k = 2[/tex] --- scale factor

See comment for proper format of question

Required

Compare FH to F'H'

From the question, we understand that the scale of dilation from EFGH to E'F'G'H is 2;

Irrespective of the center of dilation, the distance between corresponding segment will maintain the scale of dilation.

i.e.

[tex]|F'H'| = k * |FH|[/tex]

[tex]|F'H'| = 2 * |FH|[/tex]

To prove this;

Calculate distance of segments FH and F'H' using:

[tex]d = \sqrt{(x_1 - x_2)^2 + (y_1 - y_2)^2}[/tex]

Given that:

[tex]F = (1,1)[/tex]             [tex]F' = (1,2)[/tex]

[tex]H = (0,0)[/tex]            [tex]H' = (-1,0)[/tex]

We have:

[tex]FH = \sqrt{(1- 0)^2 + (1- 0)^2}[/tex]

[tex]FH = \sqrt{(1)^2 + (1)^2}[/tex]

[tex]FH = \sqrt{1 + 1}[/tex]

[tex]FH = \sqrt{2}[/tex]

Similarly;

[tex]F'H' = \sqrt{(1 --1)^2 + (2 -0)^2}[/tex]

[tex]F'H' = \sqrt{(2)^2 + (2)^2}[/tex]

Distribute

[tex]F'H' = \sqrt{(2)^2(1 +1)}[/tex]

[tex]F'H' = \sqrt{(2)^2*2}[/tex]

Split

[tex]F'H' = \sqrt{(2)^2} *\sqrt{2}[/tex]

[tex]F'H' = 2 *\sqrt{2}[/tex]

[tex]F'H' = 2\sqrt{2}[/tex]

Recall that:

[tex]|F'H'| = 2 * |FH|[/tex]

So, we have:

[tex]2\sqrt 2 = 2 * \sqrt 2[/tex]

[tex]2\sqrt 2 = 2\sqrt 2[/tex] --- true

Hence, the dilation relationship between FH and F'H' is::

[tex]|F'H'| = 2 * |FH|[/tex]

Answer:NOTT !!  A segment in the image has the same length as its corresponding segment in the pre-image.

Step-by-step explanation:

What is the range of possible sizes for side x? Please help!

Answers

Answer:

x is smaller than 5.6 and greater than 0

The product of 86 and the depth of the river

Answers

Answer:

Step-by-step explanation:

Are you trying to find a variable expression? the product of 86 means multiplication so 86*n or 86n. Other than that I dont understand the question.

The starting salaries of individuals with an MBA degree are normally distributed with a mean of $40,000 and a standard deviation of $5,000. What percentage of MBA's will have starting salaries of $34,000 to $46,000

Answers

Answer:

The correct answer is "76.98%".

Step-by-step explanation:

According to the question,

⇒ [tex]P(34000<x<46000) = P[\frac{34000-40000}{5000} <\frac{x- \mu}{\sigma} <\frac{46000-40000}{5000} ][/tex]

                                       [tex]=P(-1.2<z<1.2)[/tex]

                                       [tex]=P(z<1.2)-P(z<-1.2)[/tex]

                                       [tex]=0.8849-0.1151[/tex]

                                       [tex]=0.7698[/tex]

or,

                                       [tex]=76.98[/tex]%

Please help …………………….

Answers

9514 1404 393

Answer:

  (-3, 3)

Step-by-step explanation:

The blanks are trying to lead you through the process of finding the point of interest.

__

The horizontal distance from T to S is 9 . (or -9, if you prefer)

The ratio you're trying to divide the line into is the ratio that goes in this blank:

Multiply the horizontal distance by 2/3 . (9×2/3 = 6)

Move 6 units left from point T.

The vertical distance from T to S is 6 .

Multiply the vertical distance by 2/3 . (6×2/3 = 4)

Move 4 units up from point T.

__

Point T is (3, -1) so 6 left and 4 up is (3, -1) +(-6, 4) = (3-6, -1+4) = (-3, 3). The point that is 2/3 of the way from T to S is (-3, 3).

lisa used 880g of a container of sugar to bake a cake and 1/10 of the creaming sugar to make cookies. She then had 3/7 of the container of sugar left. How much sugar was in the container at first

Answers

Answer:

At the beginning, there were 2,678.26 grams of sugar in the container.

Step-by-step explanation:

Since Lisa used 880g of a container of sugar to bake a cake and 1/10 of the creaming sugar to make cookies, and she then had 3/7 of the container of sugar left, to determine how much sugar was in the container at first, the following calculation must be performed:

880 + 1 / 10X = 3 / 7X

880 + 0.1X = 0.4285X

880 = 0.4285X - 0.1X

880 = 0.3285X

880 / 0.3285 = X

2,678.26 = X

Therefore, at the beginning there were 2,678.26 grams of sugar in the container.

Muhammad lives twice as far from the school as Hita. Together, the live a total of 12 km
from the school. How far away drom the school does each of them live?

Answers

Answer:

Muhammad lives 8 km away from the school.

Hita lives 4 km away from the school.

Step-by-step explanation:

First of all, find a number that, when you double that number and add both numbers, you will get 12. That number is 4. So double 4 and get 8. Then add both to get 12.

An absolute value function has
A. Curved lines that only increases and decreases.
B. Straight lines that do both increase ,decrease, or stay constant on the same graph
C.Straight line that do both increase and decrease on the same graph
D. Straight lines that only increase or decrease
E. Curved lines that do both increase and decrease on the same graph

Answers

C, absolute function is basically a V shaped graph.

Samir estimates the value of Three-fifths times 16.1. Which estimate is reasonable?

3
9
12
15

Answers

Answer: 9

Step-by-step explanation:

[tex] \frac{3}{5} \times 16.1 = 9.66[/tex]

what is 24 subtracted from 8

Answers

Hi!

8 - 24 = -(24 - 8) = -16

Answer:

-16

Step-by-step explanation:

8-24=-16

Find the probability of 3 success for the binomial experiment with 7 trial and the success probability of 0.3. Then find the mean and standard deviation. Write the formula substitute
the values.

Answers

Answer:

[tex]P(x=3)=0.2269[/tex]

Mean=2.1

Standard deviation=1.21

Step-by-step explanation:

We are given that

n=7

Probability of success, p=0.3

q=1-p=1-0.3=0.7

We have to find the probability of 3 success for the binomial experiment  and find the mean and standard deviation.

Binomial distribution formula

[tex]P(X=x)=nC_xp^{x}q^{n-x}[/tex]

Using the formula

[tex]P(x=3)=7C_3(0.3)^3(0.7)^{7-3}[/tex]

[tex]P(x=3)=7C_3(0.3)^3(0.7)^{4}[/tex]

[tex]P(x=3)=\frac{7!}{3!4!}(0.3)^3(0.7)^{4}[/tex]

[tex]P(x=3)=\frac{7\times 6\times 5\times 4!}{3\times 2\times 1\times 4!}(0.3)^{3}(0.7)^{4}[/tex]

Using the formula

[tex]nC_r=\frac{n!}{r!(n-r)!}[/tex]

[tex]P(x=3)=0.2269[/tex]

Now,

Mean, [tex]\mu=np=7\times 0.3=2.1[/tex]

Standard deviation, [tex]\sigma=\sqrt{np(1-p)}[/tex]

Standard deviation, [tex]\sigma=\sqrt{7\times 0.3\times 0.7}[/tex]

Standard deviation, [tex]\sigma=1.21[/tex]

solve above question​

Answers

The probability of getting head is 3/4 or simply 0.5

The following data points represent the number of remote controllers each student in Tria's video game club owns.

Sort the data from least to greatest.

0

0

7

7

4

4

2

2

0

0

1

1

8

8

0

0

10

2

2

5

5

Find the interquartile range (IQR) of the data set.

Answers

Answer:
Data set (Least -> greatest)
0,0,0,0,0,0,1,1,2,2,2,2,4,4,5,5,7,7,8,8,10

IQR = 6

Students were sampled in order to determine their support for the legalization of gambling in their community. A sample of 150 students were asked whether or not they supported legalization of gambling, and the following results were obtained.

Do You Support? Number Of
YES 40
NO 60
NO OPINION 50

a. The value of the chi-square test statistic equals _____?
b. The number of degrees of freedom associated with this scenario is _____?

Answers

Answer:The correct answer is  They wanted to impede the sale of alcohol.

Step-by-step explanation:

They had beliefs that alcohol was against Christianity and that it ruins families and since it ruins families it should be prohibited. They eventually managed to win enough support and ban all alcohol which lasted for a few years before the prohibition ended.

Write each set in the indicated form.
If you need to use "
…" to indicate a pattern, make sure to list at least the first four elements of the set.

Answers

Answer:

a. Set-builder form: {y | y is a natural number and 12 ≤ y ≤ 15}

Or

{y | y is a natural number and 11 < y < 16}

b. Rooster form: {3, 4, 5 ,6, ...}

Step-by-step explanation:

a. Rooster form: {12, 13, 14, 15}

All four numbers are natural numbers, therefore we would write this set of numbers in set builder form such that they will all have the same property. Thus:

Set-builder form: {y | y is a natural number and 12 ≤ y ≤ 15}

Or as

{y | y is a natural number and 11 < y < 16}

b. Set-builder form: {y | y is a natural number and y > 2}

Since natural numbers are positive integers, this tells us that all values of the set are not less than or equal to 2. Therefore, they are integers that range from 3 and above.

Thus:

Rooster form: {3, 4, 5 ,6, ...}

Y+10 like terms from expression 2

Answers

Answer:

y+10=2

y=-8

Step-by-step explanation:

y=2-10

y=-8

21 × 6 ÷ 7 + 12 - 15​

Answers

Answer:

15

Step-by-step explanation:

By order of operations, multiplication and division are done first, then the addition and subtraction. Remember, multiplication and division have the same precedence, as does addition and subtraction.

21*6 = 126

126/7 = 18

18 + 12 = 30

30 - 15 = 15

Answer:

15

Step-by-step explanation:

21 × 6 ÷ 7 + 12 - 15​

= 126 ÷ 7 + 12 - 15

= 18 + 12 - 15

= 30 - 15

= 15

Two balls are picked at random from a box containing 5 red balls and 3 green balls. What is the probability that 1 red ball and 1 green ball are selected?

Answers

Answer:

Step-by-step explanation:

Answer:

3/8 x 5/8= 15/64

Step-by-step explanation:

I need a fully completed (or at least for the 8th grade) khan academy account!!!
Please help me!!

Answers

Answer:

so you need a khan academy account?

Find the length of a side of a cubic die, if the volume of the die is 343/4 cubic inches
O 1.25 in.
0 1.50 in.
0 1.75 in.
01.95 in.

Answers

Answer:

The length of a side of a cubic die of volume equal to 343/4 is 4.41 inches.

Step-by-step explanation:

The volume of a cube is given by:

[tex] V = l^{3} [/tex]

Where:

l: is the length =?  

V: is the volume = 343/4 in³

By solving the above equation for "l" we have:

[tex] l = (V)^{1/3} = (343/4 in^{3})^{1/3} = 4.41 in [/tex]

Therefore, the length of a side of a cubic die of volume equal to 343/4 is 4.41 inches.

None of the options are correct for the given volume.

I hope it helps you!                                

Use the graph of the function y=g(x) below to answer the questions.

Answers

Answer:

Step-by-step explanation:

g(5) = 2 > 0

:::::

g(x) = 0 for x = -2, 2, 4

:::::

g(x) < 0 for  -3 ≤ x < -2

If computers sell for ​$1160 per unit and hard drives sell for ​$ 102 per​ unit, the revenue from x computers and y hard drives can be represented by what​ expression? If computers sell for ​$ per unit and hard drives sell for ​$102 per​ unit, the revenue from x computers and y hard drives can be represented by

Answers

The answer to this questions is c

The profit, in dollars, of selling n items is given by P(n) = 0.86n - 2800. Identify the slope and the y-intercept.

Answers

Answer: 0.86 and -2800 (choice A)

Explanation:

Think of the given equation as y = 0.86x - 2800

Then compare it to y = mx + b

We see that m = 0.86 is the slope and b = -2800 is the y intercept.

Answer:

Slope: 0.86 , Y-intercept:-2800

Step-by-step explanation:

Linear equations go by the form of y=mx + c

where m is the gradient(slope of the graph) and c is the y-intercept

What proportion of the students scored at least 23 points on this test, rounded to five decimal places

Answers

This question is incomplete, the complete question is;

The distribution of scores on a recent test closely followed a Normal Distribution with a mean of 22 points and a standard deviation of 2 points. For this question, DO NOT apply the standard deviation rule.

What proportion of the students scored at least 23 points on this test, rounded to five decimal places?

Answer:

proportion of the students that scored at least 23 points on this test is 0.30850

Step-by-step explanation:

Given the data in the question;

mean μ = 22

standard deviation σ = 2

since test closely followed a Normal Distribution

let

Z = x-μ / σ      { standard normal random variable ]

Now, proportion of the students that scored at least 23 points on this test.

P( x ≥ 23 ) = P( (x-μ / σ) ≥  ( 23-22 / 2 )

= P( Z ≥ 1/2 )

= P( Z ≥ 0.5 )

= 1 - P( Z < 0.5 )

Now, from z table

{ we have P( Z < 0.5 ) = 0.6915 }

= 1 - P( Z < 0.5 ) = 1 - 0.6915 = 0.30850

P( x ≥ 23 ) = 0.30850

Therefore, proportion of the students that scored at least 23 points on this test is 0.30850

Please help with this function problem

Answers

Answer:

-2

-1

-2

Step-by-step explanation:

really ? this is a problem ? why ?

f(0) means the functional value for x = 0.

is x = 2 ? no.

so, automatically the other case applies, and f(0) = -2

f(2) means x=2

is x = 2 ? yes.

so that case applies, and f(2) = -1

f(5) means x=5

is x = 2 ? no.

so again, the case for x <> 2 applies, f(5) = -2

If P(x) = 2x2 – 3x + 7 and Q(x) = 8 - x), find each function value.
15. P(-3)
16. Q(2)
17. P(4)
18. Q(-3)

Answers

Answer:

15. 52

16. 6

17. 59

18. 11

Step-by-step explanation:

What is the area of triangle ABC? Round to the nearest whole number

Answers

9514 1404 393

Answer:

  C.  837

Step-by-step explanation:

The remaining angle is ...

  C = 180° -A -B = 180°-62° -67° = 51°

The law of sines tells us that the length AC is ...

  AC/sin(B) = AB/sin(C)

  AC = AB·sin(B)/sin(C) = 40·sin(67°)/sin(51°)

Using the area formula given, we now have ...

  area = 1/2(AB)(AC)sin(A)

  = (1/2)(40)(40·sin(67°)sin(62°)/sin(51°) ≈ 836.7

The area of the triangle is about 837 square units.

A new car costs $23000. The value decreases by 15% each year.(a) Write the exponential model to represent the cars value after t years. (b) To the nearest dollar, how much will the car be worth after 4 years?

Answers

Answer:

(a) 23000(1-15%)^t

(b) about 12006.14375

Step-by-step explanation:

(a) There's a formula for this problem y = A(d)^t where, A is the initial value you are given, d is the growth or decay rate and t is the time period. So, in this case, as the car cost is decreasing it is a decay problem and we can write the formula as such; y = A(1-R)^t

And with the values, we get the exponential model 23000(1-15%)^t

(b) From question (a) we already have the model and the time period given here is 4 years. So putting it in the formula we get,

23000(1-15%)^4

=23000(1-15/100)^4

=23000(0.85)^4

=23000x0.52200625

=12006.14375     (Ans)

Other Questions
Write the names of any four newly established universities of Nepal and mention the role they can play for transforming society. PLS HELP ME ON TTHIS QUESTION I WILL MARK YOU AS BRAINLIEST IF YOU KNOW THE ANSWER PLS GIVE ME A STEP BY STEP EXPLANATION WHILE YOU ANSWER THE QUESTION!!Which of the following is not a step in creating a box plot?A. Order the dataB. Find the values for the five-number summaryC. Create a number lineD. Add all of the numbers in the data set and divide by the number of data items in the set The sum of a rational number and an irrational number is irrational.Always true,sometimes true, never true Rationalize the denominator and simplify: a) (3 - 2)/( 3+2) b) (5+23)/(7+43) c) (1+2)/(3 - 22) I will give brainliest if you answer properly. (This is for other people with this Question i hope you find this when you need help) Need answer for this Help!!! Question 7 of 20 You plan to use a slingshot to launch a ball that has a mass of 0.025 kg. You want the ball to accelerate straight toward your target at 19 m/s2. How much force do you need to apply to the ball? O A. 19.03 N OB. 0.48 N O C. 4.51 N D. 760.00 N Question 3 of 10What is the value of p?V14014090-A. 50B. 90C. 60D. 40 Please help?You are trying to teach your friend how to complete a push up with proper form. Why is proper body alignment important? What are two tips you would give them about body alignment to ensure their form is correct? Se puede decir que el hombre actual, hace el uso del mimesis cmo la plante Aristteles? Y si no es as, por qu no? b. The role of the moon is greater than that of the sun in the occurrence of tides. ??? Samantha bought m candies at the store. There are n candies in a pound, and each pound costs c dollars. Write an expression for how much Samantha paid. Solve the inequality 8 + n > 4. 5 less than three times a number is 37 what is the number The nucleus of the cell is similar to the_______ system of the body. What is the most important limitation of working with stone as a building material? What detail in the excerpt supports the idea that Elizabeth is also biased against Darcy?A. My faults, according to this calculation, are heavy indeedB. These bitter accusations might have been suppressedC. Had not your pride been hurt by my honest confessionD. I thank you for explaining it so fully a person lifts 60kg on the surface of the earth, how much mass can he lift on the surface of the moon if he applies same magnitude of force In a math class of 28 students, 14 boys and 14 are girls. On a unit test, 5 boys and 9 girls made an A grade. A student is chosen at random from the class. What is the probability of choosing a girl or an A student?A. 0.82B. 0.68C. 0.14D. 0.50 If the lengths of the legs of a right triangle are 5 and 12, what is the length of the hypotenuse? Hi! Question 2-10